Continuity of Max Function in R^2

In summary, In order to approximate max(x,y) using ε delta, you need to choose δ such that ||(x,y) - (x0,y0)|| < δ. Thanks for the help!
  • #1
ttzhou
30
0
This is my first post on PF, I've been a "Google lurker" for ages though, love the quality of the help provided here. I've done a search and found similar questions for when f, g are uniformly continuous and max(f,g) is discussed, but this question is purely for (x,y) in R^2. So hopefully, I haven't tread too heavily on my first post.

I tried typing LaTeX but it's been a nightmare, hopefully I'll work out the kinks in the future.

Homework Statement



Given h : R2 ---> R, h(x,y) = max(x,y), show that h is continuous at any point (x0,y0) in its domain.


Homework Equations



There is an equation that relates max(x,y) to a function purely of the two variables, x and y. However, for the assignment, as well as my own curiosity, I wonder how one would go about doing this using the pure epsilon delta method for proving continuity, avoiding any characterization of continuity through sequences, etc.

The Attempt at a Solution



I've broken it down into cases.

1. If x0 = y0, then simply choose ||(x,y) - (x0,y0)|| < [tex]\delta = \epsilon[/tex].

Then, we note that, WLOG (we can interchange x0 and y0)

|max(x,y) - x0| <= ||(x,y) - (x0,x0)|| = ||(x,y) - (x0,y0)|| < [tex]\delta = \epsilon[/tex]

and so it is approximated by arbitrary epsilon whenever the delta condition is satisfied.


2. If x0 does not equal y0, this is where I get stumped.

I suppose WLOG that x0 < y0. (Again, I can switch around for the other case.)

Then |max(x,y) - max(x0,y0)| = |max(x,y) - y0|

(Denote M = max(x,y) for convenience)

I cannot for the life of me figure out the proper algebra to get to a nice form where I can apply the appropriate deltas. There are too many messy attempts that I've made to list all here.

I tried |M - y0| < |M - y| + |y - y0| by triangle inequality, but that keeps leaving a mismatched x and y0 or y and x0, and I can't see how finding lower/upper bounds would work here either.

My professor has stated that I am on the right track with the cases, and gave a hint about using epsilon delta to play around with the inequality x0 < y0, but I can't figure it out.

I sketched the function mentally, it looks kind of like the sharp corner of the eaves of a house; I can see it geometrically, but am unable to find the right algebraic relation.

ANY help or hints are greatly appreciated! Thank you all in advance.
 
Last edited:
Physics news on Phys.org
  • #2
welcome to pf!

hi ttzhou ! welcome to pf! :smile:

(have a delta: δ and an epsilon: ε and try using the X2 and X2 icons just above the Reply box :wink:)
ttzhou said:
Given h : R^2 ---> R, h(x,y) = max(x,y), show that h is continuous at any point (x0,y0) in its domain.

2. If x0 does not equal y0, this is where I get stumped …

yes 1 is ok (though a little messy) :smile:

for 2, just make sure your ε is less than |x0 - y0| :wink:
 
  • #3


tiny-tim said:
hi ttzhou ! welcome to pf! :smile:

yes 1 is ok (though a little messy) :smile:

Messy as in the proof itself or just the way I typed it? I tried typing [tex]\delta[/tex] and [tex]\epsilon[/tex], but it kept showing up as ||(x,y) - (x0,y0)|| [sic], just in LaTeX font.

Thanks for the tips, I'll take a crack at it later today.

EDIT: I'm having confusion as to what you mean by making [tex]\epsilon[/tex] < |x0 - y0|... Isn't [tex]\epsilon[/tex] reliant on [tex]\delta[/tex] and the point in question?
 
Last edited:
  • #4
ttzhou said:
Messy as in the proof itself … ?

Yes :redface:

i] you don't choose (x,y), you choose δ (for any ε)

ii] then you should say for any ||(x,y) - (x0,x0)|| < δ ("for any" is the opposite of choosing, isn't it? :wink:)

iii] and you should have stuck to (x0,x0) (not (x0,y0)) throughout that proof :smile:
 
  • #5
tiny-tim said:
Yes :redface:

i] you don't choose (x,y), you choose δ (for any ε)

This is my fault, I meant to say given ε > 0, choose δ = ε so that...

ii] then you should say for any ||(x,y) - (x0,x0)|| < δ ("for any" is the opposite of choosing, isn't it? :wink:)

whenever this condition is satisfied, the resulting approximation using ε follows.

iii] and you should have stuck to (x0,x0) (not (x0,y0)) throughout that proof :smile:

So I should have only changed it at the last line, since x0 = y0?


Also, still having trouble with the hint... How do I make ε less than something; isn't the approximation of ε dependent on what δ I choose?
 
  • #6
ttzhou said:
Also, still having trouble with the hint... How do I make ε less than something; isn't the approximation of ε dependent on what δ I choose?

yes, but you can choose ε to depend on δ and to be less than |x0 - y0| :smile:
 
  • #7
tiny-tim said:
yes, but you can choose ε to depend on δ and to be less than |x0 - y0| :smile:

We are given an arbitrary ε, and we must pick a δ such that the function is within ε of the image of (x0,y0)... Maybe it's the Tuesday afternoon haze, but I am just stumped as to how I can ensure ε to be less than anything, when we are only considering arbitrary values.
 
  • #8
sorry, i got my δ and my ε mixed up …

i meant make sure your δ is less than |x0 - y0| …

you can choose δ to depend on ε and to be less than |x0 - y0| :smile:
 
  • #9
I may have arrived at a solution, but it seems unelegant:

Again, assume that x0 < y0 => 0 < y0 - x0

I choose δ = min { y0 - x0, (ε/4) }

Now, if ||(x,y) - (x0, y0)|| < δ, it must be the case that:

|x - x0| < δ and |y - y0| < δ

---

This is where I find it sort of inelegant, as I essentially prove that:

x0 - y0 < x - x0 < y0 - x0 (by absolute value)

2x0 - y0 < x < y0

2x0 - 2y0 < x - y0 < 0

2(x0 - y0) < x - y0 < 2(y0 - x0)

--> |x - y0| < |2(y0 - x0)|

----

From here, I note that, denoting M = max(x,y):

|M - y0|

<= |x - y0| + |y - y0|

<= |2(y0 - x0)| + |y - y0|

< 2(ε/4) + (ε/4)

< ε

#


Does this look right? I just have my reservations about the inequality proof, I suppose I am just being an aesthetic-lover there though.
 
Last edited:
  • #10
ttzhou said:
I may have arrived at a solution, but it seems unelegant:

Again, assume that x0 < y0 => 0 < y0 - x0

I choose δ = min { y0 - x0, (ε/4) }

yeees … i think you meant "inelegant"! :redface:

the whole point of choosing that δ is so that you can start by saying that max(x,y) = y :wink:

carry from there :smile:
 
  • #11
I think what you mean is that pick a disk small enough so that any co-ordinate in that disk has the property that it lies in the region (y > x)...? But how does picking [tex]\delta[/tex] to be less than |x0 - y0| achieve this?
 
  • #12
ttzhou said:
I think what you mean is that pick a disk small enough so that any co-ordinate in that disk has the property that it lies in the region (y > x)...? But how does picking [tex]\delta[/tex] to be less than |x0 - y0| achieve this?

on second thought, it needs to be < |x0 - y0|/√2 …

draw (x0,y0), and also (y0,y0) on the line x = y …

then a circle of radius δ around (x0,y0) stays in the y>x region :wink:
 
  • #13
Sigh... the funny thing is, I originally thought of a geometric approach, but never carried it through... It would have saved me so much time.

Yea, I was able to pinpoint that exact length using some Grade 7 geometry :redface:

Thanks a lot tiny-tim, really appreciate it!

P.S... how does one develop the ability to spot elegant solutions? I always have this tendency to bull my way through problems; they work, but I miss easier ways.
 
  • #14
ttzhou said:
Sigh... the funny thing is, I originally thought of a geometric approach, but never carried it through... It would have saved me so much time.

yup! :biggrin:
P.S... how does one develop the ability to spot elegant solutions? I always have this tendency to bull my way through problems; they work, but I miss easier ways.

always draw a diagram first … these proofs are all about points and circles anyway :wink:
 

Related to Continuity of Max Function in R^2

1. What is the definition of continuity of a max function in R^2?

Continuity of a max function in R^2 refers to the property of a function where small changes in the input result in small changes in the output. In other words, as the input values approach a certain point, the output values also approach a certain value without any sudden jumps or breaks.

2. How is continuity of a max function in R^2 different from continuity in one variable?

In one variable, continuity refers to the property of a function where there are no sudden jumps or breaks in the graph. However, in R^2, a max function is continuous if it is continuous in each variable separately. This means that the function must be continuous in both the x and y directions.

3. Can a max function in R^2 be continuous at a point but not differentiable?

Yes, it is possible for a max function in R^2 to be continuous at a point but not differentiable. This can happen when the function has a sharp corner or point at that particular point, making it impossible to find a derivative.

4. What are the necessary conditions for a max function in R^2 to be continuous?

The necessary conditions for a max function in R^2 to be continuous are that the function must be defined at the point in question, the limit of the function as the input approaches the point must exist, and the value of the function at the point must be equal to the limit.

5. How can we prove the continuity of a max function in R^2?

To prove the continuity of a max function in R^2, we need to show that it satisfies the three necessary conditions: being defined at the point, having a limit at the point, and having the value at the point equal to the limit. This can be done by using the epsilon-delta definition of continuity or by using the intermediate value theorem.

Similar threads

Replies
1
Views
553
  • Programming and Computer Science
2
Replies
36
Views
3K
  • Calculus and Beyond Homework Help
Replies
27
Views
802
  • Calculus and Beyond Homework Help
Replies
22
Views
471
  • Advanced Physics Homework Help
Replies
2
Views
853
  • Calculus and Beyond Homework Help
Replies
4
Views
1K
Replies
1
Views
661
Replies
4
Views
2K
  • MATLAB, Maple, Mathematica, LaTeX
Replies
18
Views
3K
  • Calculus and Beyond Homework Help
Replies
23
Views
2K
Back
Top